Solve for x. Round to the nearest tenth of a degree, if necessary.

Solve For X. Round To The Nearest Tenth Of A Degree, If Necessary.

Answers

Answer 1

Answer:

49.6°

Step-by-step explanation:

tangent = opposite / adjacenttan x = 40/34x = arctan (40/34)x ≈ 49.6°
Answer 2

[tex] \rm \color{fuchsia}{Answer:}[/tex]

49. 6°

Step-by-step explanation:

[tex] \rm{tangent \: = \: opposite \:/ adjacent} \\ \rm{tan \: x \: = \: 40 \:/ 34} \\ \rm \: x \: = arctan \: (40 \:/ 34) \\ \: \rm {x \: = \boxed{49. \: 60°}}[/tex]


Related Questions

NEED HELPPPP RNNNNNNNN​

Answers

put it together first so
x+12-5x=3 solve from there it easy

**Does anybody knows how to do it***

instructions: state what additional information is required in order to know that the triangle are congruent for the given reason.

Given: ASA

Answers

With ASA, we need two angles and an included side. An included side is one that is between the two angles.

But, we need to make sure the sides match up. So, triangle KJI is congruent to triangle ZYX.

Therefore, we need to know that side JI is congruent to YX.

Hope this helps!

If anyone can help with this ill mark Brainly

Answers

Answer:

In this case, we can do substitution.

Step-by-step explanation:

For the first one, (s - t)(x) = ((x - 5) - 4x^2)(x) = x^2 - 21x

For the second one, (s*t)(x) = ((x - 5) *4x^2)(x) = 4x^4 - 20x^3

And for the last one,  (s+t)(-2) = ((x - 5) + 4x^2)(-2) = -8x^2 - 2x + 10

Hope your happy with the answer :)

Hello, Brainly community!

This question is for all of those Calculus people out there.

The volume of a swimming pool is changing with respect to time, such that the volume is given by W(t), where W(t) is measured in cubic centimeters and t is measured in seconds. A tangent line is shown for W(t) at t = 3 seconds. Determine the best estimate for the value of the instantaneous rate of change of W(t) when t = 3.
(I've narrowed down the answer choices to 2, and just really need to find the right way of thinking to find the answer)

(A) W(lim t) as t goes to 3.
(B) [W(3.1) - W(2.9)] / 0.2.

Thank you in advance!

Answers

Answer:

(B)  [tex]\displaystyle \frac{W(3.1) - W(2.9)}{0.2}[/tex]

General Formulas and Concepts:

Calculus

Limits

Derivatives

The definition of a derivative is the slope of the tangent line.

Derivative Notation

Instantaneous Rates

Tangent Line: [tex]\displaystyle f'(x) = \frac{f(b) - f(a)}{b - a}[/tex]

Step-by-step explanation:

Since we are trying to find a rate at which W(t) changes, we must find the derivative at t = 3.

We are given 2 close answer choices that would have the same numerical answer but different meanings:

(A)  [tex]\displaystyle \lim_{t \to 3} W(t)[/tex](B)  [tex]\displaystyle \frac{W(3.1) - W(2.9)}{0.2}[/tex]

If we look at answer choice (A), we see that our units would simply just be volume. It would not have the units of a rate of change. Yes, it may be the closest numerically correct answer, but it does not tell us the rate at which the volume would be changing and it is not a derivative.

If we look at answer choice (B), we see that our units would be cm³/s, and that is most certainly a rate of change. Answer choice (B) is also a derivative at t = 3, and a derivative tells us what rate something is changing.

∴ Answer choice (B) will give us the best estimate for the value of the instantaneous rate of change of W(t) when t = 3.

Topic: AP Calculus AB/BC (Calculus I/I + II)

Unit: Differentiation

Book: College Calculus 10e

a bag contains 3 white balls and 4 red balls. A.second bag contains 4 white balls and 3 red balls. If a bag is picked at random from each bag. Find the probability that both are white

Answers

Answer:

12/49 chance that they are both white

Step-by-step explanation:

We have 2 events going on:

The first bag that contains: 3 white, 4 red

The second bag that contains: 4 white, 3 red

First let's find the probability of picking a white ball from each individual bag (not combing the chances yet):

For the 1st bag, there are 3 white balls out of 7 balls (add 3 and 4).

That means there's a 3/7 chance you'll get a white ball from the 1st bag.

For the 2nd bag, there are 4 white balls out of 7 balls (add 4 and 3).

That means there's a 4/7 chance you'll get a white ball from the 2nd bag.

To find the probability of getting both white from both of the bags, multiply 3/7 & 4/7:

3/7 * 4/7 = 12/49

You can't simplify 12/49 further, so that's your answer!

Hope it helps (●'◡'●)

Answer:

2/6

Step-by-step explanation:

_______________________________hope it helps________

the answer is... 2/6

find the gradients of line a and b

Answers

Answer:

Gradient of A: 2

Gradient of B: -1

Step-by-step explanation:

Gradient = change in y/change in x

✔️Gradient of A using two points on line A, (2, 5) and (0, 1):

Gradient = (1 - 5)/(0 - 2) = -4/-2

Simplify

Gradient of A = 2

✔️Gradient of B using two points on line B, (0, 5) and (5, 0):

Gradient = (0 - 5)/(5 - 0) = -5/5

Simplify

Gradient of B = -1

do linear relationships have a constant rate of change ​

Answers

Answer:

Yes my son they do so yeah

Step-by-step explanation:

Cuz I know

How many black berry trees were sampled to create this graph

Answers

Answer:

C. 31 trees

Step-by-step explanation:

Add up all the frequencies: 3+3+8+10+5+2=31

Hope this helps!!!

Answer:

It's C. 31 trees

Step-by-step explanation:

That's about all I can say

Which of the following is not a way to represent the solution of the inequality 2(x − 1) greater than or equal to −12? (1 point)

x greater than or equal to −5
x less than or equal to −5
−5 less than or equal to x
A number line with a closed circle on negative 5 and shading to the right

Answers

Answer:

x less than or equal to -5

Step-by-step explanation:

2(x-1) ≥ -12

2x - 2 ≥ -12

2x ≥ -10

x ≥ -5

What is the area of a cross section that is parallel to face BFGC?

Answers

Answer: 216cm2

Step-by-step explanation: 36x6

WILL GIVE BRAINLIEST IF CORRECT!

Write an equation that represents the line. Use exact numbers.

Answers

Answer:

y=0.5x-3.5

Step-by-step explanation:

(-4)-(-5)=1

3-1=2

1/2=0.5

y=0.5x-3.5

Answer:

[tex]y=-2y-7[/tex]

Step-by-step explanation:

Coordinates plotted on the equation:

[tex](1,-4)[/tex]

[tex](4,-5)[/tex]

{check the image for your reference}

A line with a slope of 3 passes through the point (-1, 2).

Write an equation for this line in point-slope form.

Answers

Answer:

Step-by-step explanation:

Slope = m = 3

(x₁ , y₁) = (-1 , 2)

Point slope form: y - y₁ = m(x - x₁)

y - 2 = 3(x - [-1] )

y - 2 = 3(x + 1)

y - 2 = 3*x + 3*1

y - 2 = 3x + 3

    y = 3x + 3 + 2

    y = 3x + 5

Which is the area between the x-axis and y=x from x=1 to x=5

Answers

Answer:

[tex]\displaystyle A = 12[/tex]

General Formulas and Concepts:

Pre-Algebra

Order of Operations: BPEMDAS

BracketsParenthesisExponentsMultiplicationDivisionAdditionSubtractionLeft to Right

Algebra I

FunctionsFunction NotationGraphing

Calculus

Integrals

Definite IntegralsArea under the curve

Integration Rule [Reverse Power Rule]:                                                               [tex]\displaystyle \int {x^n} \, dx = \frac{x^{n + 1}}{n + 1} + C[/tex]

Integration Rule [Fundamental Theorem of Calculus 1]:                                     [tex]\displaystyle \int\limits^b_a {f(x)} \, dx = F(b) - F(a)[/tex]

Area of a Region Formula:                                                                                     [tex]\displaystyle A = \int\limits^b_a {[f(x) - g(x)]} \, dx[/tex]

Step-by-step explanation:

Step 1: Define

Identify

y = x

Interval: x = 1 to x = 5

Step 2: Sort

Graph the function. See Attachment.

Bounds of Integration: [1, 5]

Step 3: Find Area

Substitute in variables [Area of a Region Formula]:                                   [tex]\displaystyle A = \int\limits^5_1 {x} \, dx[/tex][Integral] Integrate [Integration Rule - Reverse Power Rule]:                     [tex]\displaystyle A = \frac{x^2}{2} \bigg| \limits^5_1[/tex]Evaluate [Integration Rule - Fundamental Theorem of Calculus 1]:           [tex]\displaystyle A = 12[/tex]

Topic: AP Calculus AB/BC (Calculus I/I + II)

Unit: Integration

Book: College Calculus 10e

Quadrilateral A B C D is shown. The uppercase right angle, angle A, is 79 degrees.
What are the remaining angle measures if the figure is to be a parallelogram?

m∠B =
°

m∠C =
°

m∠D =
°

Answers

Answer:

m∠B =  

✔ 101

°

m∠C =  

✔ 79

°

m∠D =  

✔ 101

°

Step-by-step explanation:

Answer:

The answer above is right!

The correct answers are:

First box: option C. 101

Second box: option B. 79

Third box: option C. 101

Step-by-step explanation:

Just got it right on edge - Hope it helps :)

Brainliest would be greatly appreciated :D

Determine the measure of <0

20.21°
0.005
73.74°
16.26°

Answers

Answer:

16.26°

Step-by-step explanation:

1. tanΘ= 7/24

2.[tex]tan^-1(\frac{7}{24} ) =[/tex]Θ

3. Θ = 16.26°

Help me to find the product plz (opt math)

Answers

Answer:

hope it helps.stay safe healthy and happy...

Answer:

[tex]\left(sin\theta -cos\:a\right)\left(cos\:a+sin\theta \right)[/tex]

(sin(θ)-cos(a))(cos(a)+sin(0))

[tex]\mathrm{Apply\:Difference\:of\:Two\:Squares\:Formula:\:}\left(a-b\right)\left(a+b\right)=a^2-b^2[/tex]

a=sin(θ),b=cos(a)

= sin²(θ)-cos²(a)

-------------------------------

hope it helps...

have a great day!!

Can someone help me with this math homework please!

Answers

Step-by-step explanation:

these are almost the simplest possible problems to get you a little bit familiar with the use of this line of thinking and to use these methods.

come on ! are you really not understanding this ?

I think you are not trying to learn and get better, you try to get others doing your work for you.

these problems are so simple that stating them here costs you more time than just solving them.

remember fractions ?

remember the problems to find the smallest common denominator for a list of fractions ?

the first question is exactly that.

what is the smallest common denominator of 3/4, 1/3 and 1/2 ?

because that is the number you need to multiply both sides to eliminate all fractions.

so, what is the smallest number that can be split by 4, 3 and 2 ?

is it 2 ? no, 4 and 3 can't do that.

is it 3 ? no, 2 and 4 can't do that.

...

is it 6 ? no, 4 can't do that.

...

is it 8 ? no, 3 can't do that.

...

is it 12 ? yeah ! all 3 can do that.

here a little tip : you start by multiplying all involved numbers, and then work your way back by eliminating some factors shared by some of the numbers.

here it would be

2×3×4 = 24

but 12 is sufficient, because 2 and 4 share another common factor of 2. so, we can divide 24 by 2 and get 12.

"isolate the variable" means having it only on one side of the equation.

so, what do you think ?

subtracting any constants does not change anything for "j".

from all the offered options only the subtraction of 1.1j eliminates j from one side.

the runners save.

you only need to translate the words into the corresponding mathematical symbols.

runner 1 has $112 already saved.

that is our starting value for this runner.

so, 112 + ...

received a $45 gift card. so, $45 are added.

so, 112 + 45 + ...

saves $25 each month (m). so, whatever the number of m is, there will be $25×m added to the account.

so, 112 + 45 + 25m

we know already, it is the second option.

but for training purposes let's look at runner 2 too :

he starts with $50 already saved.

so, 50 + ...

saves $60 each month (m). same principle as above.

so, 50 + 60m

if we want to know, when (after how many months) both runners will have the same amount of money, we simply need to say both equations are equal and solve for m.

-3x + 1 + 10x = x + 4

how do we solve this ?

by bringing all x to one side and contract them into one expression. and try to bring all constants to the other side and contact them too. and then do the reverse operation of the x expression (like a division for a multiplication).

1 + 7x = x + 4

1 + 6x = 4

6x = 3

x = 3/6 = 1/2

hurray, it is the first option !

Drag


the yellow point until an accurate "height" of the triangle is drawn. Afterwards, fill out the empty boxes below to determine the area of the triangle.

Answers

Answer:

I don't see a yellow point

Which of the equations below have the decimals aligned correctly? Check all that apply.
3.012
+ 4.21
6.71
+ 1.40
12.315
- 0.129
32.002
12.32

Answers

Answer:

B and C

Step-by-step explanation:

Decimals are well aligned when performing arithmetic operations if the equations are arranged in such a way that the decimal points are exactly vertically arranged, each following each other. This will ensure the right answer is gotten. Each place value is placed on the same vertical line on each other.

The equation in option B is correctly aligned. Because the decimal points are in the same vertical line and also the equation in option C.

what do you understand by socialization​

Answers

Socialization is the lifelong process through which people learn the values and norms of a given society.

URGENT PLS ANSWER QUICKLY​

Answers

Answer:

9th is 44 mark me as brainlist

Answer:

Question 9 is perimeter

Graph the solution of the inequality 3/7(35x-14)<_ 21x/2+3

Answers

Answer:

You'll have a closed circle at x = 2, and shading to the left

See the diagram below

=========================================================

Explanation:

The fractions here are 3/7 and 21/2. The denominators of which are 7 and 2 respectively. The LCD is 7*2 = 14.

If we multiply both sides by 14, then this will clear out the denominators and make the fractions go away.

14*(3/7) = (14*3)/7 = 42/7 = 814*(21/2) = (14*21)/2 = 294/2 = 147

So if we multiplied both sides by 14, then we have these steps

[tex]\frac{3}{7}(35x-14) \le \frac{21x}{2}+3\\\\14*\frac{3}{7}(35x-14) \le 14*\left(\frac{21x}{2}+3\right)\\\\14*\frac{3}{7}(35x-14) \le 14*\left(\frac{21x}{2}\right)+14*\left(3\right)\\\\6(35x-14) \le 147x+42\\\\[/tex]

--------------------------

Let's isolate x

[tex]6(35x-14) \le 147x+42\\\\6(35x)+6(-14) \le 147x+42\\\\210x-84 \le 147x+42\\\\210x-147x \le 42+84\\\\63x \le 126\\\\x \le 126/63\\\\x \le 2\\\\[/tex]

The graph of this will consist of a closed or filled in circle at x = 2. We shade to the left to represent numbers smaller than 2.

So either x = 2 or x < 2.

If we used an open hole at 2, then we wouldn't be including 2 (but we want to include this endpoint).

See the diagram below.

There is $1.90 in a jar filled with
quarters, dimes, and nickels.
There are 2 more quarters than
dimes and there are 2 more
nickels than quarters.
How many of each coin are there?

Answers

Answer:

7 nickels, 5 quarters, 3 dimes

Step-by-step explanation:

7 nickels= 35 cents

5 quarters= $1.25

3 dimes= 30 cents

35+ 1.25+ 30= $1.90

Hope this helps!

Plz mark Brainliest if u can :)

Identify the parts of the following algebraic expression.

-8z + 1
2
y - 7.7

Term:
Variable:
Coefficient:
Constant:

Answers

Answer:

-8z+1

term:2

variable:z

coefficient:-8

constant:1

2

term:1

variable:nil

coefficient:nil

constant:2

y-7.7

term:2

variable:y

coefficient:nil

constant:-7.7

The solution is given below.

What is number?

A number is a mathematical object used to count, measure, and label. The original examples are the natural numbers 1, 2, 3, 4, and so forth. Numbers can be represented in language with number words.

now, we get,

-8z+1

term:2

variable: z

coefficient:-8

constant:1

again,

2

term:1

variable : nil

coefficient : nil

constant:2

now,

y-7.7

term:2

variable : y

coefficient : nil

constant:-7.7

To learn more on number click:

brainly.com/question/17429689

#SPJ2

Question: Dentify the parts of the following algebraic expression.

-8z + 1

2

y - 7.7

Term:

Variable:

Coefficient:

Constant:

Can someone help with problems fivethroughseven

Answers

Answer:

5) 15120

6) 11880

7) 336

Step-by-step explanation:

The formula for permutation where mPn is m!/(m-n)!

Applying this to question 5, we get 9!/4!, which is 15120.

For question 6, we get 12!/8!, which is 11880.

For question 7, we get 8!/5!, which is 336.

Which of the following recursive formulas represents the same arithmetic sequence as the explicit formula an = 5+ (n-1)2?

Answers

Answer:

[tex]a_1 = 5[/tex]

[tex]a_n = a_{n-1} + 2[/tex]

Step-by-step explanation:

Given

[tex]a_n = 5 + (n - 1)2[/tex]

Required

The equivalent recursive function

The general explicit function is:

[tex]a_n = a_1+ (n - 1)d[/tex]

So, by comparison

[tex]a_1 = 5[/tex]

[tex]d = 2[/tex]

The recursion of an arithmetic sequence is:

[tex]a_n = a_{n-1} + d[/tex]

Substitute 2 for d

[tex]a_n = a_{n-1} + 2[/tex]

Hence: (a) is correct

CAN SOMEBODY PLEASE HELP MEEEE

Answers

Answer:

105.

Step-by-step explanation:

.

PLZ HELP its due soon!!!

Answers

Answer:

E)

Step-by-step explanation:

1) 6.50 (starting with his current wage)

2) 6.50 + 0.25 = 6.75 $

3) 6.75 + 0.25 = 7.00 $

4) 7.00 + 0.25 = 7.25 $

5) 7.25 + 0.25 = 7.50 $

6) 7.50 + 0.25 = 7.75 $

Help!!!
Find the domain of the function. Write the answer in interval notation.

Answers

Answer:

A

Step-by-step explanation:

Given

f(x) = [tex]\frac{2}{3x+2}[/tex]

The denominator cannot be zero as this would make f(x) undefined.

Equating the denominator to zero and solving gives the value that x cannot be.

3x + 2 = 0 ⇒ 3x = - 2 ⇒ x = - [tex]\frac{2}{3}[/tex] ← excluded value

Then

domain is ( - ∞ , - [tex]\frac{2}{3}[/tex] ) U ( - [tex]\frac{2}{3}[/tex], ∞ ) → A

Sierra king is a nail technician she allows 20 minutes for a manicure and 45 for a pedicure in her 7 hour work day no more than 5 pedicures can be scheduled each day the prices are $18 for a Medicare and $45 for a pedicure how many manicures and pedicures should ms.king schedule to maximize her daily income? what is her maximum daily income?​

Answers

Answer: the income is 369$

Step-by-step explanation:

Other Questions
Ozone pollution would be most favored in:_________a. sunny climates during the winter when volatile organic compounds can react with NO. b. sunny climates in summer when coal is burned to generate electric power. c.sunny climates during the summer when volatile organic compounds can react with NO. d. sunny climates in winter when coal is burned to generate electric power. what are the basic units of a chemical element called Health education play a significant role to bring about change in people's attitude and behaviour towards health Select the correct answer.How does a proper warm up affect blood flow?A.Blood flow increases.B.Blood flow decreases.C.Blood flow increases and then decreases.D.Blood flow decreases and then increases. What is the symbol for an ion with a 3+ charge, 28 electrons, and a mass number of 71 what principle did Martin Luther King jr. Was born from the political strategy of Gandhi? the place of development of society diffres from society to society.Clarify with its example 3. oraB. Busca la definicin que corresponde a cada una de las palabras. Luego escribeuna oracin completa con cada palabra. .1. olaa. concavidad formada en la tierra2. hondab. primera letra del alfabeto; preposicin que se usa paraexpresar la idea de movimiento4. holac. palabra empleada como saludod. que tiene mucha profundidad6. ollae. ola del mar: ondulacin del cabello7. haf. forma del verbo que significa rezar8. ondag. forma del verbo haberh. vasija que se usa para cocinar10. horai. unidad de tiempo equivalente a 60 minutosj. onda en el mar o un lago5. a9. hoya Work out the area of the shaded shape. compare and contrast the modern age with the primitive age Jill has just moved to a new town and wants to share plates of baked goods with her neighbors. She has 20 cookies and 12 brownies to share, and wants to split them equally among the plates with no food left over. What is the greatest number of plates she can make to share? A heel travels 850 miles in 28 gallons of gas. How many miles does it travel in one gallon of gas Please help Ladder question!!A 6 ft ladder, resting against a wall, begins to slip down the wall. When the angle of the ladder is 45 degrees, the bottom of the ladder is moving away from the wall at 0.5 m/s. At that moment, how fast is the top of ladder moving down the wall? Thomas McDonald earns $12,550 every month and sets aside the minimum amount for saving. How much would Thomas McDonaldbe saving from his income?1. $677.52. $1,2553 .$2,5104. $3,765 The line y = 2x + 6 cuts the x-axis at A and the y-axis at B. Find(a) the length of AB, (b) the shortest distance of O to AB, where O is the origin (0,0) Youre taking a walk on a warm fall morning. The temperature is about 70 degrees Fahrenheit, and you cannot see a cloud anywhere in the sky. You notice that the ground has no dew on it either. This means the 6ooo kg into quintal Building self-esteem means thinking positively about yourself balance the following half equation. SO3^2- SO4^2- What is the sum and classification of StartFraction 3 Over 20 EndFraction + StartRoot 10 EndRoot?3.31227766..., irrational3.31227766..., rational18.16227766..., irrational18.16227766..., rational